matheraum.de
Raum für Mathematik
Offene Informations- und Nachhilfegemeinschaft

Für Schüler, Studenten, Lehrer, Mathematik-Interessierte.
Hallo Gast!einloggen | registrieren ]
Startseite · Forum · Wissen · Kurse · Mitglieder · Team · Impressum
Forenbaum
^ Forenbaum
Status Mathe
  Status Schulmathe
    Status Primarstufe
    Status Mathe Klassen 5-7
    Status Mathe Klassen 8-10
    Status Oberstufenmathe
    Status Mathe-Wettbewerbe
    Status Sonstiges
  Status Hochschulmathe
    Status Uni-Analysis
    Status Uni-Lin. Algebra
    Status Algebra+Zahlentheo.
    Status Diskrete Mathematik
    Status Fachdidaktik
    Status Finanz+Versicherung
    Status Logik+Mengenlehre
    Status Numerik
    Status Uni-Stochastik
    Status Topologie+Geometrie
    Status Uni-Sonstiges
  Status Mathe-Vorkurse
    Status Organisatorisches
    Status Schule
    Status Universität
  Status Mathe-Software
    Status Derive
    Status DynaGeo
    Status FunkyPlot
    Status GeoGebra
    Status LaTeX
    Status Maple
    Status MathCad
    Status Mathematica
    Status Matlab
    Status Maxima
    Status MuPad
    Status Taschenrechner

Gezeigt werden alle Foren bis zur Tiefe 2

Navigation
 Startseite...
 Neuerdings beta neu
 Forum...
 vorwissen...
 vorkurse...
 Werkzeuge...
 Nachhilfevermittlung beta...
 Online-Spiele beta
 Suchen
 Verein...
 Impressum
Das Projekt
Server und Internetanbindung werden durch Spenden finanziert.
Organisiert wird das Projekt von unserem Koordinatorenteam.
Hunderte Mitglieder helfen ehrenamtlich in unseren moderierten Foren.
Anbieter der Seite ist der gemeinnützige Verein "Vorhilfe.de e.V.".
Partnerseiten
Dt. Schulen im Ausland: Mathe-Seiten:Weitere Fächer:

Open Source FunktionenplotterFunkyPlot: Kostenloser und quelloffener Funktionenplotter für Linux und andere Betriebssysteme
StartseiteMatheForenReelle Analysis mehrerer VeränderlichenLAGRANGE-Multiplikatoren
Foren für weitere Schulfächer findest Du auf www.vorhilfe.de z.B. Geschichte • Erdkunde • Sozialwissenschaften • Politik/Wirtschaft
Forum "Reelle Analysis mehrerer Veränderlichen" - LAGRANGE-Multiplikatoren
LAGRANGE-Multiplikatoren < mehrere Veränderl. < reell < Analysis < Hochschule < Mathe < Vorhilfe
Ansicht: [ geschachtelt ] | ^ Forum "Reelle Analysis mehrerer Veränderlichen"  | ^^ Alle Foren  | ^ Forenbaum  | Materialien

LAGRANGE-Multiplikatoren: Vorschlag
Status: (Frage) beantwortet Status 
Datum: 19:41 Do 08.01.2015
Autor: Marie886

Aufgabe
Ermitteln Sie mithilfe der Methode der LAGRANGE- Multiplikatoren die kritischen Punkte der Funktion f(x,y,z)= xyz unter der Nebenbedingung [mm] x^2+y^2+9z^2=16, [/mm] x>0, y>0, z>0

Hallo,

hab grad keinen Plan, wie ich die 4 Gleichungen auflöse kann, damit ich auf ein Ergebnis komme:

Meine Rechenschritte:

1.) Die Nebenbedingung wird zu  [mm] x^2+y^2+9z^2-16 [/mm] =0
    und Voraussetzung ist [mm] \begin{pmatrix} g_x \\ g_y \\ g_z \end{pmatrix} \ne \begin{pmatrix} 0 \\ 0 \\ 0 \end{pmatrix} =\begin{pmatrix} 2x \\ 2y \\ 18z \end{pmatrix}\ne [/mm] 0

2.) F(x,y,z)= f(x,y,z)+ [mm] \lambda [/mm] * g(x,y,z) wobei [mm] F_x=F_y=F_z=F_\lambda=0 [/mm]

            = xyz + [mm] \lambda [/mm] * [mm] (x^2+y^2+9z^2-16) [/mm]
            = xyz + [mm] \lambda x^2+\lambda y^2+\lambda 9z^2-16\lambda [/mm]

3.) Nun die Partiellen Ableitungen:

[mm] F_x:yz+2x\lambda=0 [/mm]
[mm] F_y:xz+2y\lambda=0 [/mm]
[mm] F_z:xy+18z\lambda=0 [/mm]
[mm] F_\lambda: x^2+y^2+9z^2-16=0 [/mm]

4.) An dieser Stelle würde ich die ersten 3 Gleichungen nach
[mm] \lambda [/mm] auflösen, diese dann gleichsetzen und anschließend in die 4.te Gleichung einsetzen. So habe ich es zumindest gemacht, aber ich kam leider auf kein brauchbares Ergebnis

Hat wer einen anderen Vorschlag wie ich das angehen kann??

LG

        
Bezug
LAGRANGE-Multiplikatoren: Antwort
Status: (Antwort) fertig Status 
Datum: 19:57 Do 08.01.2015
Autor: MathePower

Hallo Marie886,

> Ermitteln Sie mithilfe der Methode der LAGRANGE-
> Multiplikatoren die kritischen Punkte der Funktion
> f(x,y,z)= xyz unter der Nebenbedingung [mm]x^2+y^2+9z^2=16,[/mm]
> x>0, y>0, z>0
>  Hallo,
>  
> hab grad keinen Plan, wie ich die 4 Gleichungen auflöse
> kann, damit ich auf ein Ergebnis komme:
>  
> Meine Rechenschritte:
>
> 1.) Die Nebenbedingung wird zu  [mm]x^2+y^2+9z^2-16[/mm] =0
>      und Voraussetzung ist [mm]\begin{pmatrix} g_x \\ g_y \\ g_z \end{pmatrix} \ne \begin{pmatrix} 0 \\ 0 \\ 0 \end{pmatrix} =\begin{pmatrix} 2x \\ 2y \\ 18z \end{pmatrix}\ne[/mm]
> 0
>  
> 2.) F(x,y,z)= f(x,y,z)+ [mm]\lambda[/mm] * g(x,y,z) wobei
> [mm]F_x=F_y=F_z=F_\lambda=0[/mm]
>  
> = xyz + [mm]\lambda[/mm] * [mm](x^2+y^2+9z^2-16)[/mm]
>              = xyz + [mm]\lambda x^2+\lambda y^2+\lambda 9z^2-16\lambda[/mm]
>  
> 3.) Nun die Partiellen Ableitungen:
>  
> [mm]F_x:yz+2x\lambda=0[/mm]
>  [mm]F_y:xz+2y\lambda=0[/mm]
>  [mm]F_z:xy+18z\lambda=0[/mm]
> [mm]F_\lambda: x^2+y^2+9z^2-16=0[/mm]
>  
> 4.) An dieser Stelle würde ich die ersten 3 Gleichungen
> nach
> [mm]\lambda[/mm] auflösen, diese dann gleichsetzen und
> anschließend in die 4.te Gleichung einsetzen. So habe ich
> es zumindest gemacht, aber ich kam leider auf kein
> brauchbares Ergebnis
>  
> Hat wer einen anderen Vorschlag wie ich das angehen kann??
>  


Löse die ersten 3 Gleichungen nach x,y,z auf und setze jede dieser
Lösungen in die 4. Gleichung ein.


> LG


Gruss
MathePower

Bezug
                
Bezug
LAGRANGE-Multiplikatoren: Frage (beantwortet)
Status: (Frage) beantwortet Status 
Datum: 20:20 Do 08.01.2015
Autor: Marie886

Gut, danke!

Das habe ich nun gemacht und komme auf folgendes:

I) yz+ [mm] 2x\lambda [/mm] =0
    [mm] 2x\lambda= [/mm] -yz
     x= - [mm] \bruch{yz}{2\lambda} [/mm]

II) xz + [mm] 2y\lambda=0 [/mm]
    [mm] 2y\lambda= [/mm] -xz
    y= - [mm] \bruch{xz}{2\lambda} [/mm]

III)xy + [mm] 18z\lambda=0 [/mm]
    [mm] 18z\lambda= [/mm] -xy
    z= - [mm] \bruch{xy}{18\lambda} [/mm]



in IV einsetzen   [mm] x^2+y^2+9z^2-16=0 [/mm]
                    [mm] (-\bruch{yz}{2\lambda})^2+ [/mm] (- [mm] \bruch{xz}{2\lambda})^2 [/mm] + (- [mm] \bruch{xy}{18\lambda})^2-16 [/mm] =0
                     [mm] \bruch{y^2z^2}{4\lambda^2}+ \bruch{x^2z^2}{4\lambda^2} +\bruch{x^2y^2}{324\lambda^2}-16 [/mm] =0


hab leider noch immer keinen Plan, was nun gemacht gehört...



Bezug
                        
Bezug
LAGRANGE-Multiplikatoren: Antwort
Status: (Antwort) fertig Status 
Datum: 20:29 Do 08.01.2015
Autor: MathePower

Hallo Marie886,

> Gut, danke!
>  
> Das habe ich nun gemacht und komme auf folgendes:
>  
> I) yz+ [mm]2x\lambda[/mm] =0
>      [mm]2x\lambda=[/mm] -yz
>       x= - [mm]\bruch{yz}{2\lambda}[/mm]
>


[ok]

Setze jetzt diese Lösung in II) und III) ein,
um die Lösungen für y und z zu ermittteln,


> II) xz + [mm]2y\lambda=0[/mm]
>      [mm]2y\lambda=[/mm] -xz
>      y= - [mm]\bruch{xz}{2\lambda}[/mm]
>  
> III)xy + [mm]18z\lambda=0[/mm]
>      [mm]18z\lambda=[/mm] -xy
>      z= - [mm]\bruch{xy}{18\lambda}[/mm]
>  
>
>
> in IV einsetzen   [mm]x^2+y^2+9z^2-16=0[/mm]
>                      [mm](-\bruch{yz}{2\lambda})^2+[/mm] (-
> [mm]\bruch{xz}{2\lambda})^2[/mm] + (- [mm]\bruch{xy}{18\lambda})^2-16[/mm]
> =0
>                       [mm]\bruch{y^2z^2}{4\lambda^2}+ \bruch{x^2z^2}{4\lambda^2} +\bruch{x^2y^2}{324\lambda^2}-16[/mm]
> =0
>  
>
> hab leider noch immer keinen Plan, was nun gemacht
> gehört...
>  


Gruss
MathePower  

Bezug
                                
Bezug
LAGRANGE-Multiplikatoren: Frage (beantwortet)
Status: (Frage) beantwortet Status 
Datum: 21:09 Do 08.01.2015
Autor: Marie886

II) [mm] xz+2y\lambda=0 [/mm]
    [mm] -\bruch{yz}{2\lambda}*z+2y\lambda=0 [/mm]  
    [mm] -yz^2+4y\lambda^2=0 [/mm]
    [mm] -yz^2= -4y\lambda^2 [/mm]
    [mm] z^2=4\lambda^2 [/mm]
    
III)  [mm] xy+18z\lambda=0 [/mm]
    [mm] -\bruch{yz}{2\lambda}*y+18z\lambda=0 [/mm]  
    [mm] -y^2z+36z\lambda^2=0 [/mm]
    -y^2z= [mm] -36z\lambda^2 [/mm]
    [mm] y^2=36\lambda^2 [/mm]
  

OK, soweit so gut, falls ich richtig umgeformt hab. Muss ich jetzt all die [mm] \lambda- [/mm] Werte in die 4.te Gleichung einsetzen?

Bezug
                                        
Bezug
LAGRANGE-Multiplikatoren: Antwort
Status: (Antwort) fertig Status 
Datum: 17:45 Fr 09.01.2015
Autor: MathePower

Hallo Marie886,

> II) [mm]xz+2y\lambda=0[/mm]
>      [mm]-\bruch{yz}{2\lambda}*z+2y\lambda=0[/mm]  
> [mm]-yz^2+4y\lambda^2=0[/mm]
>      [mm]-yz^2= -4y\lambda^2[/mm]
>      [mm]z^2=4\lambda^2[/mm]
>      
> III)  [mm]xy+18z\lambda=0[/mm]
>      [mm]-\bruch{yz}{2\lambda}*y+18z\lambda=0[/mm]  
> [mm]-y^2z+36z\lambda^2=0[/mm]
>      -y^2z= [mm]-36z\lambda^2[/mm]
>      [mm]y^2=36\lambda^2[/mm]
>    
>
> OK, soweit so gut, falls ich richtig umgeformt hab. Muss
> ich jetzt all die [mm]\lambda-[/mm] Werte in die 4.te Gleichung
> einsetzen?


Wir haben je zwei Lösungen für y und z, das macht dann 4 Lösungspaare.
Zu jedem Lösungspaar (y,z) ist der zugehörige x-Wert zu ermitteln.

Dies setzt Du dann in die 4. Gleichung ein
und ermittelst dann den Parameter [mm]\lambda[/mm].

Ganz abgesehen davon ist x=y=z=0 auch eine Lösung dieser 3 Gleichungen,
erfüllt aber die Nebenbedingung nicht.


Gruss
MathePower

Bezug
        
Bezug
LAGRANGE-Multiplikatoren: Antwort
Status: (Antwort) fertig Status 
Datum: 07:38 Fr 09.01.2015
Autor: fred97


> Ermitteln Sie mithilfe der Methode der LAGRANGE-
> Multiplikatoren die kritischen Punkte der Funktion
> f(x,y,z)= xyz unter der Nebenbedingung [mm]x^2+y^2+9z^2=16,[/mm]
> x>0, y>0, z>0
>  Hallo,
>  
> hab grad keinen Plan, wie ich die 4 Gleichungen auflöse
> kann, damit ich auf ein Ergebnis komme:
>  
> Meine Rechenschritte:
>
> 1.) Die Nebenbedingung wird zu  [mm]x^2+y^2+9z^2-16[/mm] =0
>      und Voraussetzung ist [mm]\begin{pmatrix} g_x \\ g_y \\ g_z \end{pmatrix} \ne \begin{pmatrix} 0 \\ 0 \\ 0 \end{pmatrix} =\begin{pmatrix} 2x \\ 2y \\ 18z \end{pmatrix}\ne[/mm]
> 0
>  
> 2.) F(x,y,z)= f(x,y,z)+ [mm]\lambda[/mm] * g(x,y,z) wobei
> [mm]F_x=F_y=F_z=F_\lambda=0[/mm]
>  
> = xyz + [mm]\lambda[/mm] * [mm](x^2+y^2+9z^2-16)[/mm]
>              = xyz + [mm]\lambda x^2+\lambda y^2+\lambda 9z^2-16\lambda[/mm]
>  
> 3.) Nun die Partiellen Ableitungen:
>  
> [mm]F_x:yz+2x\lambda=0[/mm]
>  [mm]F_y:xz+2y\lambda=0[/mm]
>  [mm]F_z:xy+18z\lambda=0[/mm]
> [mm]F_\lambda: x^2+y^2+9z^2-16=0[/mm]



Multipliziere die 1. Gl. mit x, die 2. mit y und die 3. mit z.

Dann: [mm] $\lambda x^2= \lambda y^2=9 \lambda z^2$ [/mm]

Kann nun [mm] \lambda=0 [/mm] sein ? Nein, denn anderenfalls wäre x=0 oder y=0 oder z=0.

Also: [mm] \lambda \ne [/mm] 0.

Somit:  [mm] $x^2= y^2=9 z^2$. [/mm]

Und ab damit in die 4. Gleichung

FRED

>  
> 4.) An dieser Stelle würde ich die ersten 3 Gleichungen
> nach
> [mm]\lambda[/mm] auflösen, diese dann gleichsetzen und
> anschließend in die 4.te Gleichung einsetzen. So habe ich
> es zumindest gemacht, aber ich kam leider auf kein
> brauchbares Ergebnis
>  
> Hat wer einen anderen Vorschlag wie ich das angehen kann??
>  
> LG


Bezug
                
Bezug
LAGRANGE-Multiplikatoren: Frage (beantwortet)
Status: (Frage) beantwortet Status 
Datum: 14:28 Fr 09.01.2015
Autor: Marie886

1.) Die Nebenbedingung wird zu   [mm] x^2+y^2+9z^2-16 [/mm]  =0
    und Voraussetzung ist  [mm] \begin{pmatrix} g_x \\ g_y \\ g_z \end{pmatrix} \ne \begin{pmatrix} 0 \\ 0 \\ 0 \end{pmatrix} =\begin{pmatrix} 2x \\ 2y \\ 18z \end{pmatrix}\ne [/mm] 0

2.) F(x,y,z)= f(x,y,z)+  [mm] \lambda [/mm]  * g(x,y,z) wobei   [mm] F_x=F_y=F_z=F_\lambda=0 [/mm]
            = xyz +  [mm] \lambda [/mm]  *  [mm] (x^2+y^2+9z^2-16) [/mm]
            = xyz + [mm] \lambda x^2+\lambda y^2+\lambda 9z^2-16\lambda [/mm]

3.) Nun die partiellen Ableitungen:

[mm] F_x:yz+2x\lambda=0 [/mm]
[mm] F_y:xz+2y\lambda=0 [/mm]
[mm] F_z:xy+18z\lambda=0 [/mm]
[mm] F_\lambda: x^2+y^2+9z^2-16=0 [/mm]


Multipliziere die 1. Gl. mit x, die 2. mit y und die 3. mit z.

[mm] yz+2x\lambda=0 [/mm]
[mm] xyz+2x^2\lambda=0 [/mm]

[mm] xz+2y\lambda=0 [/mm]
[mm] xyz+2y^2\lambda=0 [/mm]

[mm] xy+18z\lambda=0 [/mm]
[mm] xyz+18z^2\lambda=0 [/mm]

=> [mm] 2x^2\lambda=2y^2\lambda=18z^2\lambda [/mm]    (mache ich das weil da eine Linearität vorliegt?)
      [mm] x^2\lambda=y^2\lambda=9z^2\lambda [/mm]  

da [mm] \lambda \ne [/mm] 0 => [mm] x^2=y^2=9z^2 [/mm] =>x=y=9z

Das nun in die 4.te Gleichung eingesetzt:

[mm] x^2+y^2+9z^2-16=0 [/mm]
[mm] x^2+x^2+x^2+-16=0 [/mm]
[mm] 3x^2=16 [/mm]
[mm] x^2=\bruch{16}{3} [/mm]

[mm] x=4*\wurzel{\bruch{1}{3}} [/mm]

da x=y => [mm] y=4*\wurzel{\bruch{1}{3}} [/mm]

da x= 9z =>z= [mm] 4*9*\wurzel{ \bruch{1}{3}} [/mm]
[mm] z=4*\wurzel{\bruch{81}{3}}=4*\wurzel{27} [/mm]
[mm] 4*\wurzel{9*3}=4*3*\wurzel{3}= 12*\wurzel{3} [/mm]

[mm] \wurzel{ \bruch{3}{4}} [/mm] + [mm] \bruch{3}{4} [/mm]

Somit sind meine kritischen Punkte:

N [mm] (4*\wurzel{\bruch{1}{3}},4*\wurzel{\bruch{1}{3}},4*\wurzel{3}) [/mm]

Soweit alles korrekt?

LG

  






  



Bezug
                        
Bezug
LAGRANGE-Multiplikatoren: Antwort
Status: (Antwort) fertig Status 
Datum: 18:03 Fr 09.01.2015
Autor: MathePower

Hallo Marie886,

> 1.) Die Nebenbedingung wird zu   [mm]x^2+y^2+9z^2-16[/mm]  =0
>      und Voraussetzung ist  [mm]\begin{pmatrix} g_x \\ g_y \\ g_z \end{pmatrix} \ne \begin{pmatrix} 0 \\ 0 \\ 0 \end{pmatrix} =\begin{pmatrix} 2x \\ 2y \\ 18z \end{pmatrix}\ne[/mm]
> 0
>
> 2.) F(x,y,z)= f(x,y,z)+  [mm]\lambda[/mm]  * g(x,y,z) wobei  
> [mm]F_x=F_y=F_z=F_\lambda=0[/mm]
> = xyz +  [mm]\lambda[/mm]  *  [mm](x^2+y^2+9z^2-16)[/mm]
> = xyz + [mm]\lambda x^2+\lambda y^2+\lambda 9z^2-16\lambda[/mm]
>  
> 3.) Nun die partiellen Ableitungen:
>  
> [mm]F_x:yz+2x\lambda=0[/mm]
> [mm]F_y:xz+2y\lambda=0[/mm]
>  [mm]F_z:xy+18z\lambda=0[/mm]
> [mm]F_\lambda: x^2+y^2+9z^2-16=0[/mm]
>
>
> Multipliziere die 1. Gl. mit x, die 2. mit y und die 3. mit
> z.
>  
> [mm]yz+2x\lambda=0[/mm]
> [mm]xyz+2x^2\lambda=0[/mm]
>
> [mm]xz+2y\lambda=0[/mm]
>  [mm]xyz+2y^2\lambda=0[/mm]
>  
> [mm]xy+18z\lambda=0[/mm]
> [mm]xyz+18z^2\lambda=0[/mm]
>
> => [mm]2x^2\lambda=2y^2\lambda=18z^2\lambda[/mm]    (mache ich das
> weil da eine Linearität vorliegt?)
>        [mm]x^2\lambda=y^2\lambda=9z^2\lambda[/mm]  
>
> da [mm]\lambda \ne[/mm] 0 => [mm]x^2=y^2=9z^2[/mm] =>x=y=9z
>  
> Das nun in die 4.te Gleichung eingesetzt:
>  
> [mm]x^2+y^2+9z^2-16=0[/mm]
> [mm]x^2+x^2+x^2+-16=0[/mm]
>  [mm]3x^2=16[/mm]
>  [mm]x^2=\bruch{16}{3}[/mm]
>  
> [mm]x=4*\wurzel{\bruch{1}{3}}[/mm]
>  

Hier gibt es zwei mögliche Lösugen: [mm]x=\pm4*\wurzel{\bruch{1}{3}}[/mm]


> da x=y => [mm]y=4*\wurzel{\bruch{1}{3}}[/mm]

>


Auch für y gibt es zwei mögliche Lösungen.

  

> da x= 9z => [mm]z=\wurzel{9*\bruch{16}{3}}=[/mm]
>  [mm]\wurzel{3*16}=4*\wurzel{3}[/mm]

>


Ebenfalls für z.

Das macht dann für   [mm]\lambda \not=0[/mm] 8 kritische Punkte.

  

> Somit sind meine kritischen Punkte:
>
> N
> [mm](4*\wurzel{\bruch{1}{3}},4*\wurzel{\bruch{1}{3}},4*\wurzel{3})[/mm]
>  


Das sind nicht die einzigen kritischen Punkte.

Abschließend ist noch der Fall [mm]\lambda=0[/mm] zu untersuchen.


> Soweit alles korrekt?
>  
> LG
>  


Gruss
MathePower

Bezug
                                
Bezug
LAGRANGE-Multiplikatoren: Vorschlag
Status: (Frage) beantwortet Status 
Datum: 19:59 So 08.02.2015
Autor: Marie886

Hallo,

noch mal zu dem Lagrange´schen Multiplikationsverfahren:



[mm] N_1(+4*\wurzel{\bruch{1}{3} },-4*\wurzel{\bruch{1}{3} }, [/mm] +4* [mm] \wurzel{3}, \lambda) [/mm]
[mm] N_2(-4*\wurzel{\bruch{1}{3} },+4*\wurzel{\bruch{1}{3} }, [/mm] -4* [mm] \wurzel{3}, \lambda) [/mm]
[mm] N_3(+4*\wurzel{\bruch{1}{3} },+4*\wurzel{\bruch{1}{3} }, [/mm] +4* [mm] \wurzel{3}, \lambda) [/mm]
[mm] N_4(-4*\wurzel{\bruch{1}{3} },-4*\wurzel{\bruch{1}{3} }, [/mm] -4* [mm] \wurzel{3}, \lambda) [/mm]
[mm] N_5(+4*\wurzel{\bruch{1}{3} },+4*\wurzel{\bruch{1}{3} }, [/mm] -4* [mm] \wurzel{3}, \lambda) [/mm]
[mm] N_6(-4*\wurzel{\bruch{1}{3} },+4*\wurzel{\bruch{1}{3} }, [/mm] +4* [mm] \wurzel{3}, \lambda) [/mm]
[mm] N_7(+4*\wurzel{\bruch{1}{3} },-4*\wurzel{\bruch{1}{3} }, [/mm] -4* [mm] \wurzel{3}, \lambda) [/mm]
[mm] N_8(-4*\wurzel{\bruch{1}{3} },-4*\wurzel{\bruch{1}{3} }, [/mm] +4* [mm] \wurzel{3}, \lambda) [/mm]


Wie kann ich denn dann noch [mm] \lambda [/mm] berechnen?

LG,
Marie886
  

Bezug
                                        
Bezug
LAGRANGE-Multiplikatoren: Antwort
Status: (Antwort) fertig Status 
Datum: 20:35 So 08.02.2015
Autor: MathePower

Hallo Marie686,

> Hallo,
>  
> noch mal zu dem Lagrange´schen Multiplikationsverfahren:
>  
>
>
> [mm]N_1(+4*\wurzel{\bruch{1}{3} },-4*\wurzel{\bruch{1}{3} },[/mm]
> +4* [mm]\wurzel{3}, \lambda)[/mm]
>  [mm]N_2(-4*\wurzel{\bruch{1}{3} },+4*\wurzel{\bruch{1}{3} },[/mm]
> -4* [mm]\wurzel{3}, \lambda)[/mm]
>  [mm]N_3(+4*\wurzel{\bruch{1}{3} },+4*\wurzel{\bruch{1}{3} },[/mm]
> +4* [mm]\wurzel{3}, \lambda)[/mm]
>  [mm]N_4(-4*\wurzel{\bruch{1}{3} },-4*\wurzel{\bruch{1}{3} },[/mm]
> -4* [mm]\wurzel{3}, \lambda)[/mm]
>  [mm]N_5(+4*\wurzel{\bruch{1}{3} },+4*\wurzel{\bruch{1}{3} },[/mm]
> -4* [mm]\wurzel{3}, \lambda)[/mm]
>  [mm]N_6(-4*\wurzel{\bruch{1}{3} },+4*\wurzel{\bruch{1}{3} },[/mm]
> +4* [mm]\wurzel{3}, \lambda)[/mm]
>  [mm]N_7(+4*\wurzel{\bruch{1}{3} },-4*\wurzel{\bruch{1}{3} },[/mm]
> -4* [mm]\wurzel{3}, \lambda)[/mm]
>  [mm]N_8(-4*\wurzel{\bruch{1}{3} },-4*\wurzel{\bruch{1}{3} },[/mm]
> +4* [mm]\wurzel{3}, \lambda)[/mm]
>  


Zunächst, die dritte Komponente (z-Komponente) stimmt nicht.


>
> Wie kann ich denn dann noch [mm]\lambda[/mm] berechnen?
>  


Einsetzen in eine Gleichung, die [mm]\lambda[/mm] enthält.


> LG,
>  Marie886

>


Gruss
MathePower      

Bezug
                                                
Bezug
LAGRANGE-Multiplikatoren: Frage (beantwortet)
Status: (Frage) beantwortet Status 
Datum: 21:17 So 08.02.2015
Autor: Marie886

Gut danke,

also für z kommt raus: z= [mm] 4*9*\wurzel{\bruch{1}{3}}= 4*\wurzel{\bruch{81}{3}}= 4*\wurzel{27}= 4*\wurzel{9*3}= 4*3*\wurzel{3}= 12*\wurzel{3} [/mm]    


Und um [mm] \lambda [/mm] zu erhalten habe ich die erste Gleichung verwendet:

[mm] yz+2x\lambda=0 [/mm]
[mm] \lambda= -\bruch{yz}{2x}= -\bruch{4*\wurzel{\bruch{1}{3}}*12*\wurzel{3}}{2*4*\wurzel{\bruch{1}{3}}}= -\bruch{4*\wurzel{\bruch{1}{3}}*\wurzel{\bruch{144}{3}}}{\wurzel{\bruch{64}{3}}}= -\bruch{4*\wurzel{\bruch{1}{3}}*\wurzel{48}}{\wurzel{\bruch{64}{3}}}= -\bruch{4*\wurzel{\bruch{1}{3}}*\wurzel{16*3}}{\wurzel{4*16}}= -\bruch{4*\wurzel{\bruch{1}{3}}*4\wurzel{3}}{2*4}= [/mm]
[mm] \bruch{-16*\wurzel{\bruch{3}{3}}}{8}= -2*\wurzel{1}=-2 [/mm]

Ist das jetzt so korrekt? [mm] \lambda [/mm] hat aber jetzt nur eine Lösung oder? Das negative Vorzeichen kam ja nicht von der Wurzel...

LG,
Marie886

Bezug
                                                        
Bezug
LAGRANGE-Multiplikatoren: Antwort
Status: (Antwort) fertig Status 
Datum: 21:39 So 08.02.2015
Autor: MathePower

Hallo Marie686,



> Gut danke,
>  
> also für z kommt raus: z= [mm]4*9*\wurzel{\bruch{1}{3}}= 4*\wurzel{\bruch{81}{3}}= 4*\wurzel{27}= 4*\wurzel{9*3}= 4*3*\wurzel{3}= 12*\wurzel{3}[/mm]
>    

Für z kommt heraus: [mm]z= \ \pm \bruch{4}{3\wurzel{3}}[/mm]


>
>
> Und um [mm]\lambda[/mm] zu erhalten habe ich die erste Gleichung
> verwendet:
>  
> [mm]yz+2x\lambda=0[/mm]
>  [mm]\lambda= -\bruch{yz}{2x}= -\bruch{4*\wurzel{\bruch{1}{3}}*12*\wurzel{3}}{2*4*\wurzel{\bruch{1}{3}}}= -\bruch{4*\wurzel{\bruch{1}{3}}*\wurzel{\bruch{144}{3}}}{\wurzel{\bruch{64}{3}}}= -\bruch{4*\wurzel{\bruch{1}{3}}*\wurzel{48}}{\wurzel{\bruch{64}{3}}}= -\bruch{4*\wurzel{\bruch{1}{3}}*\wurzel{16*3}}{\wurzel{4*16}}= -\bruch{4*\wurzel{\bruch{1}{3}}*4\wurzel{3}}{2*4}=[/mm]
> [mm]\bruch{-16*\wurzel{\bruch{3}{3}}}{8}= -2*\wurzel{1}=-2[/mm]
>  
> Ist das jetzt so korrekt? [mm]\lambda[/mm] hat aber jetzt nur eine
> Lösung oder? Das negative Vorzeichen kam ja nicht von der
> Wurzel...
>  


Nein, das ist nicht korrekt.

Natürlich musst Du alle ermittelten Werte nacheinander
in diese Gleichung einsetzen. Dann erhältst Du auch
verschiedene [mm]\lambda[/mm].


> LG,
>  Marie886


Gruss
MathePower

Bezug
                                                                
Bezug
LAGRANGE-Multiplikatoren: Frage (beantwortet)
Status: (Frage) beantwortet Status 
Datum: 14:13 Mo 09.02.2015
Autor: Marie886

Meine Rechenschritte:

1.) Die Nebenbedingung wird zu   [mm] x^2+y^2+9z^2-16 [/mm]  =0
    und Voraussetzung ist  [mm] \begin{pmatrix} g_x \\ g_y \\ g_z \end{pmatrix} \ne \begin{pmatrix} 0 \\ 0 \\ 0 \end{pmatrix} =\begin{pmatrix} 2x \\ 2y \\ 18z \end{pmatrix}\ne [/mm] 0

2.) F(x,y,z)= f(x,y,z)+  [mm] \lambda [/mm]  * g(x,y,z) wobei  [mm] F_x=F_y=F_z=F_\lambda=0 [/mm]

            = xyz +  [mm] \lambda [/mm]  *  [mm] (x^2+y^2+9z^2-16) [/mm]
            = xyz +  [mm] \lambda x^2+\lambda y^2+\lambda 9z^2-16\lambda [/mm]

3.) Nun die Partiellen Ableitungen:

I:   [mm] F_x:yz+2x\lambda=0 [/mm]
II:  [mm] F_y:xz+2y\lambda=0 [/mm]
III: [mm] F_z:xy+18z\lambda=0 [/mm]
IV:  [mm] F_\lambda: x^2+y^2+9z^2-16=0 [/mm]

I auf x umformen:   yz+  [mm] 2x\lambda [/mm]  =0
                    [mm] 2x\lambda= [/mm] -yz
                    x= - [mm] \bruch{yz}{2\lambda} [/mm]

x in II einsetzen: [mm] xz+2y\lambda=0 [/mm]
                   [mm] -\bruch{yz}{2\lambda}\cdot{}z+2y\lambda=0 [/mm]  
                   [mm] -yz^2+4y\lambda^2=0 [/mm]
                   [mm] -yz^2= -4y\lambda^2 [/mm]
                   [mm] z^2=4\lambda^2 [/mm]
    
x in III einsetzen:   [mm] xy+18z\lambda=0 [/mm]
                      [mm] -\bruch{yz}{2\lambda}\cdot{}y+18z\lambda=0 [/mm]  
                      [mm] -y^2z+36z\lambda^2=0 [/mm]
                      -y^2z=  [mm] -36z\lambda^2 [/mm]
                      [mm] y^2=36\lambda^2 [/mm]

Multipliziere I mit x, II mit y und III mit z:

[mm] yz+2x\lambda=0 [/mm]
[mm] xyz+2x^2\lambda=0 [/mm]

[mm] xz+2y\lambda=0 [/mm]
[mm] xyz+2y^2\lambda=0 [/mm]

[mm] xy+18z\lambda=0 [/mm]
[mm] xyz+18z^2\lambda=0 [/mm]

=> [mm] 2x^2\lambda=2y^2\lambda=18z^2\lambda [/mm]  
      [mm] x^2\lambda=y^2\lambda=9z^2\lambda [/mm]

da [mm] \lambda \ne [/mm]  0 =>  [mm] x^2=y^2=9z^2 [/mm]  =>x=y=9z

Das nun in die 4.te Gleichung eingesetzt:

[mm] x^2+y^2+9z^2-16=0 [/mm]
[mm] x^2+x^2+x^2+-16=0 [/mm]
[mm] 3x^2=16 [/mm]
[mm] x^2=\bruch{16}{3} [/mm]

x= [mm] \pm 4\cdot{}\wurzel{\bruch{1}{3}} [/mm]

somit ist [mm] x_1= [/mm] + [mm] 4\cdot{}\wurzel{\bruch{1}{3}} [/mm]
somit ist [mm] x_2= [/mm] - [mm] 4\cdot{}\wurzel{\bruch{1}{3}} [/mm]

da [mm] x^2= y^2 [/mm] => [mm] y^2= \bruch{16}{3} [/mm]

[mm] y=\pm 4\cdot{}\wurzel{\bruch{1}{3}} [/mm]

somit ist [mm] y_1= [/mm] + [mm] 4\cdot{}\wurzel{\bruch{1}{3}} [/mm]  
somit ist [mm] y_2= [/mm] - [mm] 4\cdot{}\wurzel{\bruch{1}{3}} [/mm]

da [mm] x^2=9z^2 [/mm] => [mm] z^2= \bruch{x^2}{9}= \bruch{16}{27}=> [/mm] z= [mm] \wurzel{ \bruch{16}{27} }= [/mm]
               [mm] \bruch{4}{\wurzel{9*3}}=\pm \bruch{4}{3*\wurzel{3}} [/mm]

somit ist [mm] z_1= [/mm] + [mm] \bruch{4}{3*\wurzel{3}} [/mm]
somit ist [mm] z_2= [/mm] - [mm] \bruch{4}{3*\wurzel{3}} [/mm]

Um [mm] \lambda [/mm] zu erhalten werden alle ermittelten Werte in die gleiche Gleichung eingesetzt:

yz+  [mm] 2x\lambda [/mm]  =0
[mm] 2x\lambda= [/mm] -yz
[mm] \lambda= [/mm] - [mm] \bruch{yz}{2x} [/mm]

[mm] \lambda_1= [/mm] - [mm] \bruch{y_1z_1}{2x_1}= -\bruch{4*\wurzel{\bruch{1}{3}}*\bruch{4}{3*\wurzel{3} }}{2*4\cdot{}\wurzel{\bruch{1}{3}} }= -\bruch{4*4\wurzel{\bruch{1}{3}}*\bruch{1}{3*\wurzel{3} }}{8\cdot{}\wurzel{\bruch{1}{3}} }= [/mm]
- [mm] \bruch{16* \bruch{1}{3} *\wurzel{\bruch{1}{3*3} }}{8*\wurzel{\bruch{1}{3} }}=-\bruch{2*\bruch{1}{3}* \wurzel{\bruch{1}{9} }}{\wurzel{\bruch{1}{3} }}= -\bruch{2*\bruch{1}{3} *\bruch{1}{3} }{\wurzel{\bruch{1}{3}}} [/mm] = [mm] -\bruch{2*\bruch{1}{9}}{\wurzel{\bruch{1}{3} }}= -\bruch{2*\wurzel{3}}{9} [/mm]

[mm] \lambda_2= [/mm] - [mm] \bruch{y_2z_2}{2x_2}= -\bruch{-4*\wurzel{\bruch{1}{3}}*-\bruch{4}{3*\wurzel{3} }}{-2*4\cdot{}\wurzel{\bruch{1}{3}} }= -\bruch{4*4\wurzel{\bruch{1}{3}}*\bruch{1}{3*\wurzel{3} }}{-8\cdot{}\wurzel{\bruch{1}{3}} }= [/mm]
- [mm] \bruch{16* \bruch{1}{3} *\wurzel{\bruch{1}{3*3} }}{-8*\wurzel{\bruch{1}{3} }}=\bruch{2*\bruch{1}{3}* \wurzel{\bruch{1}{9} }}{\wurzel{\bruch{1}{3} }}= \bruch{2*\bruch{1}{3} *\bruch{1}{3} }{\wurzel{\bruch{1}{3}}} [/mm] = [mm] \bruch{2*\bruch{1}{9}}{\wurzel{\bruch{1}{3} }}= +\bruch{2*\wurzel{3}}{9} [/mm]

somit ist [mm] \lambda_1=-\bruch{2*\wurzel{3}}{9} [/mm]
somit ist [mm] \lambda_2=+\bruch{2*\wurzel{3}}{9} [/mm]

und meine 8 kritischen Punkte lauten:

[mm] N_1 \left( + 4\cdot{}\wurzel{\bruch{1}{3}},+ 4\cdot{}\wurzel{\bruch{1}{3}},+ \bruch{4}{3*\wurzel{3}}, -\bruch{2*\wurzel{3}}{9} \right) [/mm]

[mm] N_2 \left( - 4\cdot{}\wurzel{\bruch{1}{3}},- 4\cdot{}\wurzel{\bruch{1}{3}},- \bruch{4}{3*\wurzel{3}}, +\bruch{2*\wurzel{3}}{9} \right) [/mm]

so und nun müssts richtig sein?!

LG,
Marie886





Bezug
                                                                        
Bezug
LAGRANGE-Multiplikatoren: Antwort
Status: (Antwort) fertig Status 
Datum: 14:26 Mo 09.02.2015
Autor: fred97


> Meine Rechenschritte:
>  
> 1.) Die Nebenbedingung wird zu   [mm]x^2+y^2+9z^2-16[/mm]  =0
>      und Voraussetzung ist  [mm]\begin{pmatrix} g_x \\ g_y \\ g_z \end{pmatrix} \ne \begin{pmatrix} 0 \\ 0 \\ 0 \end{pmatrix} =\begin{pmatrix} 2x \\ 2y \\ 18z \end{pmatrix}\ne[/mm]
> 0
>  
> 2.) F(x,y,z)= f(x,y,z)+  [mm]\lambda[/mm]  * g(x,y,z) wobei  
> [mm]F_x=F_y=F_z=F_\lambda=0[/mm]
>
> = xyz +  [mm]\lambda[/mm]  *  [mm](x^2+y^2+9z^2-16)[/mm]
> = xyz +  [mm]\lambda x^2+\lambda y^2+\lambda 9z^2-16\lambda[/mm]
>
> 3.) Nun die Partiellen Ableitungen:
>  
> I:   [mm]F_x:yz+2x\lambda=0[/mm]
> II:  [mm]F_y:xz+2y\lambda=0[/mm]
> III: [mm]F_z:xy+18z\lambda=0[/mm]
> IV:  [mm]F_\lambda: x^2+y^2+9z^2-16=0[/mm]
>
> I auf x umformen:   yz+  [mm]2x\lambda[/mm]  =0
>                      [mm]2x\lambda=[/mm] -yz
>                      x= - [mm]\bruch{yz}{2\lambda}[/mm]
>
> x in II einsetzen: [mm]xz+2y\lambda=0[/mm]
>                    
> [mm]-\bruch{yz}{2\lambda}\cdot{}z+2y\lambda=0[/mm]  
> [mm]-yz^2+4y\lambda^2=0[/mm]
> [mm]-yz^2= -4y\lambda^2[/mm]
> [mm]z^2=4\lambda^2[/mm]
>
> x in III einsetzen:   [mm]xy+18z\lambda=0[/mm]
> [mm]-\bruch{yz}{2\lambda}\cdot{}y+18z\lambda=0[/mm]  
> [mm]-y^2z+36z\lambda^2=0[/mm]
> -y^2z=  [mm]-36z\lambda^2[/mm]
> [mm]y^2=36\lambda^2[/mm]
>
> Multipliziere I mit x, II mit y und III mit z:
>  
> [mm]yz+2x\lambda=0[/mm]
>  [mm]xyz+2x^2\lambda=0[/mm]
>  
> [mm]xz+2y\lambda=0[/mm]
> [mm]xyz+2y^2\lambda=0[/mm]
>
> [mm]xy+18z\lambda=0[/mm]
> [mm]xyz+18z^2\lambda=0[/mm]
>
> => [mm]2x^2\lambda=2y^2\lambda=18z^2\lambda[/mm]  
> [mm]x^2\lambda=y^2\lambda=9z^2\lambda[/mm]
>  
> da [mm]\lambda \ne[/mm]  0 =>  [mm]x^2=y^2=9z^2[/mm]  =>x=y=9z

>  
> Das nun in die 4.te Gleichung eingesetzt:
>  
> [mm]x^2+y^2+9z^2-16=0[/mm]
> [mm]x^2+x^2+x^2+-16=0[/mm]
> [mm]3x^2=16[/mm]
> [mm]x^2=\bruch{16}{3}[/mm]
>
> x= [mm]\pm 4\cdot{}\wurzel{\bruch{1}{3}}[/mm]
>
> somit ist [mm]x_1=[/mm] + [mm]4\cdot{}\wurzel{\bruch{1}{3}}[/mm]
> somit ist [mm]x_2=[/mm] - [mm]4\cdot{}\wurzel{\bruch{1}{3}}[/mm]
>
> da [mm]x^2= y^2[/mm] => [mm]y^2= \bruch{16}{3}[/mm]
>
> [mm]y=\pm 4\cdot{}\wurzel{\bruch{1}{3}}[/mm]
>
> somit ist [mm]y_1=[/mm] + [mm]4\cdot{}\wurzel{\bruch{1}{3}}[/mm]  
> somit ist [mm]y_2=[/mm] - [mm]4\cdot{}\wurzel{\bruch{1}{3}}[/mm]
>
> da [mm]x^2=9z^2[/mm] => [mm]z^2= \bruch{x^2}{9}= \bruch{16}{27}=>[/mm] z=
> [mm]\wurzel{ \bruch{16}{27} }=[/mm]
>                
> [mm]\bruch{4}{\wurzel{9*3}}=\pm \bruch{4}{3*\wurzel{3}}[/mm]
>  
> somit ist [mm]z_1=[/mm] + [mm]\bruch{4}{3*\wurzel{3}}[/mm]
> somit ist [mm]z_2=[/mm] - [mm]\bruch{4}{3*\wurzel{3}}[/mm]
>
> Um [mm]\lambda[/mm] zu erhalten werden alle ermittelten Werte in die
> gleiche Gleichung eingesetzt:
>  
> yz+  [mm]2x\lambda[/mm]  =0
>  [mm]2x\lambda=[/mm] -yz
>  [mm]\lambda=[/mm] - [mm]\bruch{yz}{2x}[/mm]
>
> [mm]\lambda_1=[/mm] - [mm]\bruch{y_1z_1}{2x_1}= -\bruch{4*\wurzel{\bruch{1}{3}}*\bruch{4}{3*\wurzel{3} }}{2*4\cdot{}\wurzel{\bruch{1}{3}} }= -\bruch{4*4\wurzel{\bruch{1}{3}}*\bruch{1}{3*\wurzel{3} }}{8\cdot{}\wurzel{\bruch{1}{3}} }=[/mm]
>  
> - [mm]\bruch{16* \bruch{1}{3} *\wurzel{\bruch{1}{3*3} }}{8*\wurzel{\bruch{1}{3} }}=-\bruch{2*\bruch{1}{3}* \wurzel{\bruch{1}{9} }}{\wurzel{\bruch{1}{3} }}= -\bruch{2*\bruch{1}{3} *\bruch{1}{3} }{\wurzel{\bruch{1}{3}}}[/mm]
> = [mm]-\bruch{2*\bruch{1}{9}}{\wurzel{\bruch{1}{3} }}= -\bruch{2*\wurzel{3}}{9}[/mm]
>
> [mm]\lambda_2=[/mm] - [mm]\bruch{y_2z_2}{2x_2}= -\bruch{-4*\wurzel{\bruch{1}{3}}*-\bruch{4}{3*\wurzel{3} }}{-2*4\cdot{}\wurzel{\bruch{1}{3}} }= -\bruch{4*4\wurzel{\bruch{1}{3}}*\bruch{1}{3*\wurzel{3} }}{-8\cdot{}\wurzel{\bruch{1}{3}} }=[/mm]
>  
> - [mm]\bruch{16* \bruch{1}{3} *\wurzel{\bruch{1}{3*3} }}{-8*\wurzel{\bruch{1}{3} }}=\bruch{2*\bruch{1}{3}* \wurzel{\bruch{1}{9} }}{\wurzel{\bruch{1}{3} }}= \bruch{2*\bruch{1}{3} *\bruch{1}{3} }{\wurzel{\bruch{1}{3}}}[/mm]
> = [mm]\bruch{2*\bruch{1}{9}}{\wurzel{\bruch{1}{3} }}= +\bruch{2*\wurzel{3}}{9}[/mm]
>
> somit ist [mm]\lambda_1=-\bruch{2*\wurzel{3}}{9}[/mm]
> somit ist [mm]\lambda_2=+\bruch{2*\wurzel{3}}{9}[/mm]
>
> und meine 8 kritischen Punkte lauten:
>  
> [mm]N_1 \left( + 4\cdot{}\wurzel{\bruch{1}{3}},+ 4\cdot{}\wurzel{\bruch{1}{3}},+ \bruch{4}{3*\wurzel{3}}, -\bruch{2*\wurzel{3}}{9} \right)[/mm]
>  
> [mm]N_2 \left( - 4\cdot{}\wurzel{\bruch{1}{3}},- 4\cdot{}\wurzel{\bruch{1}{3}},- \bruch{4}{3*\wurzel{3}}, +\bruch{2*\wurzel{3}}{9} \right)[/mm]
>  
> so und nun müssts richtig sein?!

Ja. Aber die berechnung von [mm] \lambda [/mm] hättest Du Dir sparen können !

FRED

>  
> LG,
>  Marie886
>  
>
>
>  


Bezug
                                                                                
Bezug
LAGRANGE-Multiplikatoren: Frage (überfällig)
Status: (Frage) überfällig Status 
Datum: 14:57 Mo 09.02.2015
Autor: Marie886

Super, danke!

Wieso hätte ich mir die sparen können? Kann ich mir die Berechnung bei der Prüfung sparen oder hätte ich es mir sparen können sie im Forum zu posten?

LG,
Marie886

Bezug
                                                                                        
Bezug
LAGRANGE-Multiplikatoren: Fälligkeit abgelaufen
Status: (Mitteilung) Reaktion unnötig Status 
Datum: 15:20 Mi 11.02.2015
Autor: matux

$MATUXTEXT(ueberfaellige_frage)
Bezug
Ansicht: [ geschachtelt ] | ^ Forum "Reelle Analysis mehrerer Veränderlichen"  | ^^ Alle Foren  | ^ Forenbaum  | Materialien


^ Seitenanfang ^
www.matheraum.de
[ Startseite | Forum | Wissen | Kurse | Mitglieder | Team | Impressum ]